The resistances in the following figure are in ohm. Then the effective resistance between the points A and B is? (A) 2#Omega# (B) 3 #Omega# (C) 6#Omega# (D) 36 #Omega#

enter image source here

1 Answer
Dec 17, 2016

enter image source here

In the given network for resistor if we consider the portion ACD , we observe that across AD resistor #R_(AC)# and #R_(CD)#are in series and #R_(AD)# is parallel . So the equivalent resistance of this portion across AD becomes

#R_"eqAD"=1/(1/(R_(AC)+R_(CD))+1/R_(AD))=1/(1/((3+3))+1/6)=3Omega#

and we get equivalent network #color(red)2#

similarly if we proceed , we finally reach at figure #color(red)4# i.e.equivalent network #ABF# and the equivalent resistance of the given network across AB becomes

#R_"eqAB"==1/(1/(R_(AF)+R_(FB))+1/R_(AB))=1/(1/((3+3))+1/3)=2Omega#